Number of crimes - Assumption ques

This topic has expert replies
User avatar
Legendary Member
Posts: 1460
Joined: Tue Dec 29, 2009 1:28 am
Thanked: 135 times
Followed by:7 members

Number of crimes - Assumption ques

by selango » Wed Jun 23, 2010 3:43 am
To decrease the number of crimes in city Y,the city police commissioner proposed taking some police officers from low crime districts of the city and moving them to high crime districts of the city.His proposal is based on city Y crime data that the number of crimes in any district of the city decreases when additional police officers are moved into that district.The police commissioner's proposal depends on which of the following assumptions?

A.City X experienced a drastic reduction in crime after implementing the proposal similar to that proposed by police commissioner of city X

B.The severity of crimes committed in any district of the city decreases when additional police officers are moved into that district.

C.The number of crimes commited in all high crime districts of city Y is more than triple the number of crimes commited in all low crime district of Y.

D.there are more low crime districts than high crime districts in City Y.

E.Districts of city from which police officers are removed do not expericence significant crime shortly after the removal of those officers.

OA later

Explain ur answers.

Senior | Next Rank: 100 Posts
Posts: 76
Joined: Fri Dec 11, 2009 1:37 am

by er_priyankajolly » Wed Jun 23, 2010 4:32 am
I would go for E
conclusion is --because of movement of some additional officers crime decreases.
because of the following reasons
A) An example is put forward that something similar happened and was succesful but no mention about city X
B) Out of scope. "severity of crimes" is not the concern.Its number of crimes
C)Not relevant. We need something which says about movement helps in lowering crimes.Negating it wont effect conclusion.
D)Not relevant.Negating it wont effect the conclusion.
E)If you negate it then it hampers the conclusion.Because if additional officers descrease crime then reduction in officers should icrease crime.however the option says the it remains roughly the same.

What is the correct answer?

Legendary Member
Posts: 1119
Joined: Fri May 07, 2010 8:50 am
Thanked: 29 times
Followed by:3 members

by diebeatsthegmat » Thu Jul 01, 2010 11:34 am
selango wrote:To decrease the number of crimes in city Y,the city police commissioner proposed taking some police officers from low crime districts of the city and moving them to high crime districts of the city.His proposal is based on city Y crime data that the number of crimes in any district of the city decreases when additional police officers are moved into that district.The police commissioner's proposal depends on which of the following assumptions?

A.City X experienced a drastic reduction in crime after implementing the proposal similar to that proposed by police commissioner of city X

B.The severity of crimes committed in any district of the city decreases when additional police officers are moved into that district.

C.The number of crimes commited in all high crime districts of city Y is more than triple the number of crimes commited in all low crime district of Y.

D.there are more low crime districts than high crime districts in City Y.

E.Districts of city from which police officers are removed do not expericence significant crime shortly after the removal of those officers.

OA later

Explain ur answers.
E is my as

Senior | Next Rank: 100 Posts
Posts: 64
Joined: Tue Jan 05, 2010 4:15 am
Thanked: 1 times
Followed by:1 members

by dream700 » Fri Jul 02, 2010 1:46 am
IMO it's D.

The Police Commissioner can take out from some districts only when the crime rates are lower than those in dist Y.
And moreover addition of troops means reduction in crime rates but it doesnt mean resuction of troops will results in increased crime.

Deutssch750

Master | Next Rank: 500 Posts
Posts: 191
Joined: Sat Jun 26, 2010 9:27 am
Thanked: 6 times
Followed by:2 members

by ashish2104 » Fri Jul 02, 2010 2:08 am
IMO E.

@dream700: Option D states that, number of low crime districts is more than high crime districts. this is out of scope as number of low/ high crime districts is not discussed.

Option E correctly disccuses the assumption that moving police officers does not change the balance of crime in city Y.

User avatar
Legendary Member
Posts: 1261
Joined: Sun Sep 14, 2008 3:46 am
Thanked: 27 times
GMAT Score:570

by reply2spg » Sat Jul 24, 2010 7:50 pm
E is better for me
selango wrote:To decrease the number of crimes in city Y,the city police commissioner proposed taking some police officers from low crime districts of the city and moving them to high crime districts of the city.His proposal is based on city Y crime data that the number of crimes in any district of the city decreases when additional police officers are moved into that district.The police commissioner's proposal depends on which of the following assumptions?

A.City X experienced a drastic reduction in crime after implementing the proposal similar to that proposed by police commissioner of city X

B.The severity of crimes committed in any district of the city decreases when additional police officers are moved into that district.

C.The number of crimes commited in all high crime districts of city Y is more than triple the number of crimes commited in all low crime district of Y.

D.there are more low crime districts than high crime districts in City Y.

E.Districts of city from which police officers are removed do not expericence significant crime shortly after the removal of those officers.

OA later

Explain ur answers.
Sudhanshu
(have lot of things to learn from all of you)

User avatar
Legendary Member
Posts: 1460
Joined: Tue Dec 29, 2009 1:28 am
Thanked: 135 times
Followed by:7 members

by selango » Sun Jul 25, 2010 3:13 am
OA E
--Anand--